Jump to content

phudinhgioihan's Content

There have been 329 items by phudinhgioihan (Search limited from 05-06-2020)



Sort by                Order  

#382215 Ảnh thành viên

Posted by phudinhgioihan on 31-12-2012 - 11:15 in Góc giao lưu

Có chú nào muốn làm em rể giơ tay lên, giơ chân luôn nhé :)) Posted Image



#394100 [Giải trí]Cặp đôi hoàn hảo VMF 2013

Posted by phudinhgioihan on 06-02-2013 - 22:07 in Góc giao lưu

đẹp của đẹp luôn.Like cho anh Đạt và chị Hạ nào.Anh ơi chị này có phải là GF của anh ko


Chuẩn man :D. Được cộng nhiêu điểm vậy ta ^_^



#394088 [Giải trí]Cặp đôi hoàn hảo VMF 2013

Posted by phudinhgioihan on 06-02-2013 - 21:56 in Góc giao lưu

Đạt em tham gia rồi, cho Đạt anh đi ké :D. Mong được nhiều vote để up hình tiếp :D.

Lấy hình honey luôn vậy, có gì bị ăn đòn sau :icon6:

Posted Image

Posted Image


Trương Tấn Đạt ft Nguyễn Thị Khánh Hạ




#402583 Ôn thi Olympic Toán học sinh viên 2015 [Giải tích]

Posted by phudinhgioihan on 06-03-2013 - 21:12 in Thảo luận về các kì thi, các kì kiểm tra Toán sinh viên

Một bài có nhiều ứng dụng trong tính giới hạn liên quan đến tích phân ;
Bài 35:Cho hàm $f:[a,b]\to\mathbb{R}$ liên tục trên miền xác định với $0\leq a<b$ và cho hàm $g:[0,+\infty)\to\mathbb{R}$ liên tục ,tuần hoàn với chu kì $T$.Chứng minh rằng:$$\displaystyle\lim_{n\to+\infty}\int_a^b f(x)g(nx)dx =\dfrac{1}{T}\int_0^T g(x) \int_a^b f(x)dx$$


Hic, khế chua khế chát :(

Do $g$ liên tục trên $[a;b]$, do đó, tồn tại $M > 0$ sao cho $ g(x)+M >0 \;, \forall x \in [a;b] $

Đặt $h(x)=g(x)+M $ , khi đó $h$ liên tục dương và tuần hoàn chu kỳ $T$ trên $[a;b]$.

Với $n$ rất lớn gọi $m \in \mathbb{N}$ lớn nhất sao cho $\dfrac{mT}{n}<b-a $ , theo định lý trung bình tích phân ta có:

$$\int_a^b f(x)h(nx)=\sum_{i=0}^m \int_{a+\frac{iT}{n}}^{a+\frac{(i+1)T}{n}} f(x) h(nx)dx+\int_{a+\frac{mT}{n}}^b f(x)h(nx)dx$$

$$=\sum_{i=0}^m f(c_m )\int_{a+\frac{iT}{n}}^{a+\frac{(i+1)T}{n}} h(nx)dx+\int_{a+\frac{mT}{n}}^b f(x)h(nx)dx $$

$$=\sum_{i=0}^m f(c_m )\dfrac{1}{n} \int_{a+iT}^{a+(i+1)T} h(x)dx+\int_{a+\frac{mT}{n}}^b f(x)h(nx)dx $$

$$=\sum_{i=0}^m f(c_m )\dfrac{1}{n}( \int_{a+iT}^0 h(x)dx+\int_0^T h(x)dx+\int_T^{a+{i+1}T} h(x)dx )+\int_{a+\frac{mT}{n}}^b f(x)h(nx)dx $$

$$=\sum_{i=0}^m f(c_m )\dfrac{1}{n} \int_0^T h(x)dx+\int_{a+\frac{mT}{n}}^b f(x)h(nx)dx $$

$$=\sum_{i=0}^m f(c_m )\dfrac{T}{n} \dfrac{1}{T}\int_0^T h(x)dx +\int_{a+\frac{mT}{n}}^b f(x)h(nx)dx$$

$$=\sum_{i=0}^m f(c_m ) (a+\dfrac{(i+1)T}{n}-a-\dfrac{iT}{n}) \dfrac{1}{T}\int_0^T h(x)dx+\int_{a+\frac{mT}{n}}^b f(x)h(nx)dx $$


Suy ra

$$\lim_{n \to +\infty} \int_a^b f(x)h(nx)dx$$
$$=\lim_{n \to+\infty}\left(\sum_{i=0}^m f(c_m ) (a+\dfrac{(i+1)T}{n}-a-\dfrac{iT}{n}) \dfrac{1}{T}\int_0^T h(x)dx+\int_{a+\frac{mT}{n}}^b f(x)h(nx)dx \right)$$

$$= \dfrac{1}{T}\int_0^T h(x)dx \int_a^b f(x)dx $$

$$\Leftrightarrow \lim_{n \to +\infty} \int_a^bf(x)( g(nx)+M)dx=\dfrac{1}{T}\int_0^T (g(x)+M)dx \int_a^b f(x)dx $$

$$\Leftrightarrow \lim_{n \to +\infty} \int_a^bf(x) g(nx)dx=\dfrac{1}{T}\int_0^T g(x)dx \int_a^b f(x)dx $$



#398426 Ôn thi Olympic Toán học sinh viên 2015 [Giải tích]

Posted by phudinhgioihan on 19-02-2013 - 22:38 in Thảo luận về các kì thi, các kì kiểm tra Toán sinh viên

Bài 37:

Phát biểu vấn đề giống bài 34 :D

Cho $f$ xác định và khả vi vô hạn lần trên $(-1;1)$ sao cho $\forall k \in \mathbb{N}^*, \; f^{(k)}(0) \neq 0$. Theo ông Taylor, với $0<|x|<1$ và $n \in \mathbb{N}^*$

$f(x)=f(0)+xf'(0)+...+\dfrac{x^{n-1}f^{(n-1)}(0)}{(n-1)!}+\dfrac{x^nf^{(n)}(\theta_x x)}{n!} \;, 0<\theta_x<1 $

Tính $\lim_{x \to 0} \theta_x $



#404014 Ôn thi Olympic Toán học sinh viên 2015 [Giải tích]

Posted by phudinhgioihan on 11-03-2013 - 14:29 in Thảo luận về các kì thi, các kì kiểm tra Toán sinh viên

Bài 46: Cho $f(x)$ là một hàm liên tục trên $[0,1]$ sao co với mỗi $x\epsilon [0,1]$ thì $\int_{x}^{1}f(t)dt\geq \frac{1-x^2}{2}$
Chứng minh: $f(x) \geq x$ từ đó suy ra: $\int_{0}^{1}f(t)dt\geq \frac{1}{3}$


Bài này và bài 14 trong cùng topic này tương tự nhau , lời giải cho bài 14 đã có ở đây

http://diendantoanho...2013-giải-tich/

Chắc có nhầm lẫn hay sao á, cho $x=0$ là có ngay kết quả rồi



#407382 Ôn thi Olympic Toán học sinh viên 2015 [Giải tích]

Posted by phudinhgioihan on 24-03-2013 - 01:10 in Thảo luận về các kì thi, các kì kiểm tra Toán sinh viên

Bài 33: (mới chế :D)

Sao có cảm giác thân quen @@

Cho $[a;b] \subset \mathbb{R}$ , $f:[a;b] \to \mathbb{R}$ , $f \in C^2([a;b])$ sao cho $f(a)=f(b)$. Đặt $M=\max_{[a;b]} f''(x)\;, m=\min_{[a;b]} f''(x)$

Giả sử tồn tại $0<h<b-a$ sao cho $f(a+h)=f(b-h)$, chứng minh:

$$\left| f'(a)+f'(b) \right| \le \dfrac{M-m}{2}h$$

 

 

Lỡ ghi nhầm, giờ mới sửa lại đề, xin lỗi mọi người nhiều nhiều :( (chắc do già cả khụ khị lẩm cẩm nó thế :D)

 

Theo công thức Taylor phần dư tích phân, ta có

 

$$f(a+h)=f(a)+hf'(a)+\int_a^{a+h} (a+h-t)f''(t)dt $$

 

$$f(b-h)=f(b)-hf'(b)+\int_{b}^{b-h}(b-h-t)f''(t)dt $$

 

$$\Rightarrow f(a+h)-f(b-h)=h(f'(a)+f'(b))+\int_a^{a+h} (a+h-t)f''(t)dt+\int_{b-h}^b (b-h-t)f''(t)dt $$

 

$$ \le h(f'(a)+f'(b))+M \int_a^{a+h} (a+h-t)dt+m \int_{b-h}^b (b-h-t)dt$$

 

$$ =h(f'(a)+f'(b))+\dfrac{M-m}{2}h^2 \;\; ( *)$$

 

Tương tự thế:

 

$$f(a+h)-f(b-h)=h(f'(a)+f'(b))+\int_a^{a+h} (a+h-t)f''(t)dt+\int_{b-h}^b (b-h-t)f''(t)dt $$

 

$$\ge h(f'(a)+f'(b))+m \int_a^{a+h} (a+h-t)dt +M \int_{b-h}^b (b-h-t)dt$$

 

$$ =h(f'(a)+f'(b))+\dfrac{m-M}{2}h^2 \;\;\;, (**)$$

 

 

Từ $(*)$ và $(**)$ suy ra $|f'(a)+f'(b)| \le \dfrac{M-m}{2}h $




#463925 Ôn thi Olympic Toán học sinh viên 2015 [Giải tích]

Posted by phudinhgioihan on 12-11-2013 - 20:16 in Thảo luận về các kì thi, các kì kiểm tra Toán sinh viên


Câu 5. Cho $f(x)$ là hàm dương, liên tục trên đoạn $[0,1]$ và thỏa mãn điều kiện $ f(x)+f\left( \left( 1-\sqrt{x} \right)^2 \right) \le 1 $ với mọi $x \in [0,1]$.  Chứng minh rằng
$$ \int_0^1 \sqrt{f(x)} \, dx \le \frac{\pi\sqrt5}{8}. $$
 

 

Do sự xuất hiện của $x$ và $(1-\sqrt{x})^2$ nên nghĩ ngay tới lượng giác hóa :D

 

Bằng đổi biến $x=\cos^4 t $, ta được

 

$$\int_0^1 \sqrt{f(x)}dx=4\int_0^{\frac{\pi}{2}} \sqrt{f(\cos^4 t)}\cos^3 t \sin t dt \le^{C-S} 4 \sqrt{\int_0^{\frac{\pi}{2}} f(\cos^4 x)dx}\sqrt{\int_0^{\frac{\pi}{2}} \cos^6 x \sin^2 x dx}$$

 

Bằng đổi biến $x=\sin^4 t $

 

$$\int_0^1 \sqrt{f(x)} dx =4\int_0^{\frac{\pi}{2}}\sqrt{f(\sin^4 t)} \sin^3t \cos t dt \le^{C-S}4 \sqrt{\int_0^{\frac{\pi}{2}} f(\sin^4 x)dx} \sqrt{\int_0^{\frac{\pi}{2}} \sin^6 x \cos^2 x dx} $$

 

Bằng đổi biến $x=\frac{\pi}{2}-t$

 

$$\int_0^{\frac{\pi}{2}} \cos^6 x \sin^2 x dx=\int_0^{\frac{\pi}{2}} \sin^6 x \cos^2 x dx$$

 

 

Vì vậy

 

$$2 \left( \int_0^1 \sqrt{f(x)}dx \right)^2 \le 16 \int_0^{\frac{\pi}{2}} \left( f(\cos^4x)+f(\sin^4x) \right)dx \int_0^{\frac{\pi}{2}} \sin^6x \cos^2xdx $$

 

Theo giả thiết, $f(\cos^4x)+f(\sin^4x) \le 1 $

 

$\int_0^{\frac{\pi}{2}} \sin^6x \cos^2xdx=\dfrac{5\pi}{256}$

 

Cho nên $$\left( \int_0^1 \sqrt{f(x)}dx \right)^2 \le \dfrac{5 \pi^2}{64}$$

 

$$\Rightarrow \int_0^1 \sqrt{f(x)}dx \le \dfrac{\sqrt{5} \pi}{8}$$

 

Dấu "=" xảy ra khi và chỉ khi $f(\cos^4x)=a \cos^3x\sin x\;, f(\sin^4 x)=b\sin^3 x\cos x$ với $(a,b) \in \mathbb{R}^2 $

 

Khi đó $f(\cos^4 x)+f(\sin^4 x)=a \cos^3x\sin x+b\sin^3 x\cos x$

 

Theo giả thiết thì phải có $a \cos^3x\sin x+b\sin^3 x\cos x \le 1 \; \forall x \in \mathbb{R}$, hơn nữa
, để có dấu "=" thì $\int_0^{\frac{\pi}{2}} \left( f(\cos^4x)+f(\sin^4x) \right)dx=\dfrac{\pi}{2}$

 

Suy ra $a \cos^3x\sin x+b\sin^3 x\cos x=1 \; \forall x \in \mathbb{R}$ nhưng điều này là mâu thuẫn vì thay $x=0$ thì $a \cos^3x\sin x+b\sin^3 x\cos x=0 \neq 1 $

 

Tóm lại, ta có bất đẳng thức thật sự $$\int_0^1 \sqrt{f(x)}dx < \dfrac{\sqrt{5} \pi}{8}$$

 

 

Nhận xét: Ta có thể giải cách khác để thu được hằng số tốt hơn $\dfrac{\sqrt{5} \pi}{8}$ nhưng hơi dài nên không ghi ra ở đây :D.




#463995 Ôn thi Olympic Toán học sinh viên 2015 [Giải tích]

Posted by phudinhgioihan on 12-11-2013 - 22:03 in Thảo luận về các kì thi, các kì kiểm tra Toán sinh viên

 

Câu 6: Thí sinh chọn một trong hai câu:


b) Cho hàm số $f(x)$ liên tục trên $[0,1]$. Chứng minh rằng nếu tồn tại hàm $g(x)$ đơn điệu thực sự (tức là đơn điệu và $g(x) \ne g(y)$ nếu $x \ne y$) và liên tục trên đoạn $[0,1]$ sao cho
$$ \int_0^1 f(x)g^k(x)\,d(x)=0, \ \ \forall k=0,1,\ldots,2013 $$
thì phương trình $f(x)=0$ có ít nhất 2014 nghiệm phân biệt nằm trong khoảng $(0,1)$.
Hãy chỉ ra thí dụ nếu bỏ tính đơn điệu của hàm $g(x)$ thì định lý có thể không đúng.

 

 

Với $P \in \mathbb{R}_{2013}[x]$ là đa thức hệ số thực bậc nhỏ hơn 2013 bất kỳ, từ giả thiết, ta luôn có $\int_0^1 f(x)P(x)dx=0$

 

Giả sử phương trình $f(x)=0$ có ít hơn 2014 nghiệm trong $(0;1)$. Bỏ qua các nghiệm bội mà tại đó hàm số $f$ đổi dấu, sắp xếp các nghiệm còn lại theo thứ tự tăng dần $x_0=0 < x_1<x_2<...<x_m < 1=x_{m+1}  \;, m <2014$. Trên hai khoảng liên tiếp $(x_i;x_{i+1})$ và $(x_{i+1};x_{i+2}) \;, 0 \le i \le m-1$, hàm số $f(x)$ nhận giá trị dấu khác nhau. Đặt $h(x)=\prod_{i=1}^{m}(g(x)-g(x_i))$, do $g(x)$ đơn điệu nên trong hai khoảng liên tiếp $(x_i;x_{i+1})$ và $(x_{i+1};x_{i+2}) \;, 0 \le i \le m-1$ $h(x)$ cũng nhận giá trị dấu khác nhau. Do đó, $f(x)h(x)$ hoặc dương hoặc âm trên $[0;1]$, do đó $\int_0^1 f(x)h(x) dx \neq 0$. Tuy nhiên, $h \in \mathbb{R}_{2013}[x]$ nên theo nhận xét đầu tiên, ta phải có $\int_0^1 f(x)h(x)dx=0$. Mâu thuẫn.

 

Vậy phương trình $f(x)=0$ có ít nhất $2014$ nghiệm thực phân biệt.

 

Nếu bỏ giả thiết $g(x)$ đơn điệu. Xét $r_1(x),r_2(x)$ là hai hàm thực liên tục trên $[0;1]$ sao cho $r_1$ là hàm lẻ và $r_2$ là hàm chẵn. Đặt $f(x)=r_1(x-\frac{1}{2}) \;, g(x)=r_2(x-\frac{1}{2} )$.

 

Với $k \in \mathbb{Z}, 0 \le k \le 2013$

 

$$\int_0^1f(x)g^k(x)dx=\int_0^{\frac{1}{2}} r_1(x-\frac{1}{2})r_2^{k}(x-\frac{1}{2})dx+\int_{\frac{1}{2}}^1 r_1(x-\frac{1}{2})r_2^{k}(x-\frac{1}{2})dx$$

 

$$\int_0^{\frac{1}{2}} r_1(x-\frac{1}{2})r_2^{k}(x-\frac{1}{2})dx=^{x=1-t} \int_{\frac{1}{2}}^1r_1(\frac{1}{2}-t) r_2^k(\frac{1}{2}-t)dt $$

 

$$=-\int_{\frac{1}{2}}^1 r_1(x-\frac{1}{2})r_2^{k}(x-\frac{1}{2})dx$$

 

Do đó $\int_0^1f(x)g^k(x)dx=0$

 

Nhưng $f(x)=0 \Leftrightarrow r_1(x-\frac{1}{2})=0 $ có số nghiệm tùy thuộc vào $r_2(x)$, rõ ràng có vô số hàm số $r_2$ thỏa yêu cầu nhưng phương trình $r_2(x-\frac{1}{2}) =0$ chỉ có một nghiệm thực. Ví dụ $r_{1a}(x)=ax \;, a \neq 0$.




#463875 Ôn thi Olympic Toán học sinh viên 2015 [Giải tích]

Posted by phudinhgioihan on 12-11-2013 - 19:07 in Thảo luận về các kì thi, các kì kiểm tra Toán sinh viên


Câu 4. Cho hàm $f(x)$ liên tục trên $[0,1]$ và khả vi trong $(0,1)$ thỏa mãn $f(0)=0 ;\;  f(1)=1$. Chứng minh rằng tồn tại các số phân biệt $x_1,x_2,\ldots,x_{2013} \in (0,1)$ sao cho
$$ \sum_{k=1}^{2013} \frac{kx_k}{f'(x_k)}=\frac{2013 \times 1007}{2}. $$
 

 

Đây là bài toán ...chả biết gọi sao nữa :D, xem sau sẽ rõ ^_^

 

Tổng quát, thay $2013$ bởi $n \in \mathbb{N}^* $ cho đẹp.

 

$\dfrac{2013 \times 1007}{2} $ thay bằng $\dfrac{n(n+1)}{4}=\frac{1}{2}\left( 1+2+...+n \right)$

 

Ta cần chứng minh tồn tại $n$ số thực phân biệt trong $(0;1)$ sao cho $$\sum_{k=1}^n \dfrac{kx_k}{f'(x_k)}=\dfrac{1+2+...+n}{2}$$

 

$$\Leftrightarrow \sum_{k=1}^n \dfrac{k2x_k}{f'(x_k)}=1+2+...+n$$

 

$$\Leftrightarrow \sum_{k=1}^n \dfrac{k}{1+2...+n} \dfrac{2x_k}{f'(x_k)}=1$$

 

Biểu thức $\dfrac{2x_k}{f'(x_k)}$ gợi ý tới $\dfrac{(x^2)^\prime}{f^\prime(x)}$ tức liên quan đến định lý Cauchy.

 

Đặt $z_k=\dfrac{1+...+k}{1+...+n} \;, 1 \le k \le n-1 $, hiển nhiên $z_k \in (0;1) \;, \forall 1 \le k \le n-1 $

 

Hàm $f$ liên tục trên $[0;1]$ và $f(0)=0 <1=f(1) $ nên theo định lý giá trị trung gian, với mỗi $z_k, \; 1 \le k \le n-1$ phải tồn tại $y_k \in (0;1) $ sao cho $z_k=f(y_k)$ (chọn các $y_k$ tất cả đều khác nhau). Không mất tính tổng quát, giả sử $(y_k)_{1 \le k \le n-1}$ là dãy tăng.

 

Đặt $y_0=0,y_n=1 $

 

Áp dụng định lý Cauchy cho $n$ đoạn $[y_i; y_{i+1}] \;, 0 \le i \le n-1$ với hàm $f$ và $g(x)=x^2$, tồn tại các $x_{i+1} \in (y_i;y_{i+1}) \;, 0 \le i \le n-1$ sao cho

 

$$\dfrac{y_{i+1}^2-y_i^2}{f(y_{i+1}-f(y_i)}=\dfrac{2x_{i+1}}{f'(x_{i+1})}$$

 

$$\Leftrightarrow y_{i+1}^2-y_i^2=\dfrac{2x_{i+1}}{f'(x_{i+1})}( f(y_{i+1}-f(y_i))=\dfrac{2x_{i+1}}{f'(x_{i+1})}(z_{i+1}-z_i)=\dfrac{2x_{i+1}}{f'(x_{i+1})} \dfrac{i+1}{1+...+n}$$

 

$$\Rightarrow \sum_{i=0}^{n-1} \left( y_{i+1}^2-y_i^2 \right) =\sum_{i=0}^{n-1} \dfrac{2x_{i+1}}{f'(x_{i+1})} \dfrac{i+1}{1+...+n} $$

 

$$\Leftrightarrow 1=\sum_{i=0}^{n-1} \dfrac{2x_{i+1}}{f'(x_{i+1})} \dfrac{i+1}{1+...+n} $$

 

Đây chính là điều phải chứng minh.




#463857 Ôn thi Olympic Toán học sinh viên 2015 [Giải tích]

Posted by phudinhgioihan on 12-11-2013 - 18:12 in Thảo luận về các kì thi, các kì kiểm tra Toán sinh viên



Câu 3. Cho $\alpha \ge \beta > 0$. Hãy tìm các hàm số $f : (0, +\infty ) \to \mathbb{R}$ thỏa mãn điều kiện $ f(x) = \max \{ x^{\alpha}y^{\beta} - f(y) : y \ge x \}$ với mọi $ x \in \ (0,+\infty). $
 

 

 

Hình như đây là một bài trong đề thi ở đâu đó ( lỡ quên mất :D)

 

Với $x>0$,

 

từ giả thiết $f(x)=\max_{y \ge x} ( x^\alpha y^\beta-f(y) ) $ suy ra $f(x) \ge x^\alpha y^\beta-f(y) \;, \forall y \ge x $

 

Cho $y=x$ ta được $f(x) \ge \dfrac{x^{\alpha+\beta}}{2}$

 

Do đó $x^\alpha y^\beta-f(y) \le x^\alpha y^\beta -\dfrac{y^{\alpha+\beta}}{2} , \forall y \ge x$

 

Ta chứng minh bất đẳng thức $x^\alpha y^\beta-\dfrac{y^{\alpha+\beta}}{2} \le \dfrac{x^{\alpha+\beta}}{2} \;, \forall y \ge x$

 

$$\Leftrightarrow \left(\frac{x}{y} \right)^\beta  + \left( \frac{y}{x} \right)^\alpha \ge 2 , \; \forall y \ge x$$

 

Do $\dfrac{y}{x} \ge 1 $ và $\alpha \ge \beta >0 $ nên

 

$$\left(\frac{x}{y} \right)^\beta  + \left( \frac{y}{x} \right)^\alpha \ge \left(\frac{x}{y} \right)^\beta  + \left( \frac{y}{x} \right)^\beta \ge 2 \sqrt{\left(\frac{x}{y} \right)^\beta \left( \frac{y}{x} \right)^\beta} =2$$

 

Như vậy phải có $f(x)=\dfrac{x^{\alpha+\beta}}{2}$

 

Thử lại, hàm số $f(x)=\dfrac{x^{\alpha+\beta}}{2}$ thỏa mãn điều kiện đề bài.




#463856 Ôn thi Olympic Toán học sinh viên 2015 [Giải tích]

Posted by phudinhgioihan on 12-11-2013 - 17:54 in Thảo luận về các kì thi, các kì kiểm tra Toán sinh viên



Câu 2. Tìm giới hạn $$ \lim _{n \rightarrow \infty} \int^{1}_{0} \frac{nx^n}{2013+x^n} dx. $$

 

 

Làm biếng đóng thành file pdf nên gõ từng bài vậy :D

 

Rất nhiều người làm sai bài này khi nghĩ rằng $\lim _{n \rightarrow \infty} \int^{1}_{0} \frac{nx^n}{2013+x^n} dx=\int_0^1 \left( \lim_{n \to \infty} \dfrac{nx^n}{2013+x^n} \right) dx $

 

Khi nào $f_n $ liên tục trên $[a;b]  \;, \forall n \in \mathbb{N}$ và $(f_n)_{n \in \mathbb{N}}$ hội tụ đều về một hàm số $f$ thì khi  đó mới có thể kết luận

 

$\lim_{n \to \infty} \int_a^b f_n(x)dx=\int_a^b \left( \lim_{n \to \infty} f_n(x) \right) dx =\int_a^b f(x)dx$

 

Quay trở lại vấn đề, bản chất ở đây chỉ là giới hạn dãy số, do đó cố gắng đánh giá hàm trong tích phân để được giới hạn kẹp :D

 

$$\int_0^1 \dfrac{nx^n}{2013+x^n}dx=\left( x \ln (2013+x^n) \right)|_0^1-\int_0^1 \ln (2013+x^n)dx$$

 

$$=\ln 2014-\int_0^1 \ln 2013(1+\dfrac{x^n}{2013})dx =\ln \dfrac{2014}{2013}-\int_0^1 \ln(1+\dfrac{x^n}{2013})dx$$

 

Áp dụng bdt quen thuộc, $\forall t \ge 0,\;\; 0 \le \ln(1+t) \le t$, ta được

 

$$0 \le \ln(1+\dfrac{x^n}{2013}) \le \dfrac{x^n}{2013}  \Rightarrow 0\le \int_0^1 \ln(1+\dfrac{x^n}{2013})dx \le \int_0^1 \frac{x^n}{2013}dx=\frac{1}{2013(n+1)}$$

 

Do $\lim_{n \to \infty} \frac{1}{2013(n+1)}=0$ nên từ đây suy ra $\lim_{n \to \infty} \int_0^1 \ln(1 +\dfrac{x^n}{2013})dx=0 $

 

Vậy $$ \lim _{n \rightarrow \infty} \int^{1}_{0} \frac{nx^n}{2013+x^n} dx=\ln \dfrac{2014}{2013} $$




#394018 Ôn thi Olympic Toán học sinh viên 2015 [Giải tích]

Posted by phudinhgioihan on 06-02-2013 - 20:29 in Thảo luận về các kì thi, các kì kiểm tra Toán sinh viên

Bài 34: (B.Jacobson)
Cho $[a;b] \subset \mathbb{R}\;\;, f:[a;b] \to \mathbb{R}$ khả vi $n \in \mathbb{N^*}$ lần trên $[a;b]$ sao cho $\lim_{x \to a^+} f^{(k)}(x)=0\;, \forall k \in \{1;...;n-1\}$ và $\lim_{x \to a^+} f^{(n)}(x) \neq 0$. Theo định lý trung bình tích phân, với $x \in (a;b] , \exists c_x \in (a;x) \;, \int_a^x f(t)dt=f(c_x)(x-a)$. Chứng minh

$$\lim_{x \to a^+} \dfrac{c_x-a}{x-a}=\dfrac{1}{\sqrt[n]{n+1}}$$



#393950 Ôn thi Olympic Toán học sinh viên 2015 [Giải tích]

Posted by phudinhgioihan on 06-02-2013 - 19:25 in Thảo luận về các kì thi, các kì kiểm tra Toán sinh viên

Bài 33: (mới chế :D)

Sao có cảm giác thân quen @@

Cho $[a;b] \subset \mathbb{R}$ , $f:[a;b] \to \mathbb{R}$ , $f \in C^2([a;b])$ sao cho $f(a)=f(b)$. Đặt $M=\max_{[a;b]} f''(x)\;, m=\min_{[a;b]} f''(x)$

Giả sử tồn tại $0<h<b-a$ sao cho $f(a+h)=f(b-h)$, chứng minh:

$$\left| f'(a)+f'(b) \right| \le \dfrac{M-m}{2}h$$




#387480 Ôn thi Olympic Toán học sinh viên 2015 [Giải tích]

Posted by phudinhgioihan on 17-01-2013 - 19:55 in Thảo luận về các kì thi, các kì kiểm tra Toán sinh viên

Bài 14(olimpic sv Ngoại thương 2013) Cho hàm số $f\left(x\right)$ xác định, liên tục trên $\left[1;2\right]$ thỏa mãn $\displaystyle\int_a^b f^2\left(x\right)dx \leq \dfrac{b^3-a^3}{3},\forall a,b\in\left[1;2\right],a\leq b$.Chứng minh rằng $$\displaystyle\int_1^2 f\left(x\right)dx\leq \dfrac{3}{2}$$


Hôm nay tự dưng thông minh ra :))

Số $\dfrac{b^3-a^3}{3}$ gợi ý đến $\int_a^b x^2 dx$ , ta viết lại giả thiết

$$\int_a^b f^2(x)dx \le \int_a^b x^2dx $$
$$\Leftrightarrow \int_a^b ( f^2(x)-x^2) dx \le 0 \;\; , \forall 1 \le a<b \le 2 $$

Theo định lý giá trị trung bình tích phân, $$\forall \; 1\le a<b \le 2 \;\;, \exists c \in [a;b] \;, \; f^2( c )-c^2 \le 0$$

Hàm $h(x)=f^2(x)-x^2 $ liên tục trên $[1;2]$

Với mỗi $x_0 \in [1;2) $ , $\forall h >0 $ đủ nhỏ sao cho $x_0+h \in [1;2]$, theo trên, phải tồn tại $ x_h \in [x_0;x_0+h] \;, f^2(x_h)-x_h^2 \le 0$


Vì $f$ liên tục trên $[1;2]$ , cho $h \to 0 \Rightarrow x_h^2 \to x_0^2 \;, \; f^2(x_0+h) \to f^2(x_0)$

$$\Rightarrow f^2(x_0)-x_0^2 \le 0$$

Suy ra $$f^2(x) \le x^2 \;\; , \forall x \in [1;2) $$

Xét $x_0=2 $ , $\forall h>0 $ đủ nhỏ sao cho $2-h \in [1;2]$ , tồn tại $x_h \in [2-h;2] \;\;, f^2(x_h)-x_h^2 \le 0$

Cho $h \to 0$ ta nhận được $f^2(2)-4 \le 0$

Vậy $$\forall x \in [1;2] \;\;, f^2(x) \le x^2 $$

$$\Rightarrow f(x) \le |f(x)| \le x \;\;, \forall \; x \in [1;2] $$

$$\Rightarrow \int_1^2 f(x)dx \le \int_1^2 xdx=\dfrac{3}{2}$$

____________________________________________________________________________

P/s: Đề Olympic năm 2000 ^_^, nhưng đáp án sơ sài quá.



#385053 Ôn thi Olympic Toán học sinh viên 2015 [Giải tích]

Posted by phudinhgioihan on 09-01-2013 - 18:42 in Thảo luận về các kì thi, các kì kiểm tra Toán sinh viên

Bài 9:

Cho $f: [0;1] \longrightarrow \mathbb{R}$ , $f$ liên tục trên $[0;1]$ . Chứng minh

$$\lim_{n \to +\infty} \int_0^1 \dfrac{nf(x)dx}{1+n^2x^2}=\frac{\pi}{2}f(0) $$



#384676 Ôn thi Olympic Toán học sinh viên 2015 [Giải tích]

Posted by phudinhgioihan on 08-01-2013 - 14:40 in Thảo luận về các kì thi, các kì kiểm tra Toán sinh viên

Bài 8: ( BDT E.Landau)

a) Cho $f:[a;b] \longrightarrow \mathbb{R} $ với $a,b$ là số thực sao cho $b-a \ge 2 $, $f$ khả vi cấp 2 và thỏa $|f(x)| \le 1 \;, |f''(x)| \le 1 $

Chứng minh: $$|f'(x) | \le 2 $$

b) Cho $f$ là hàm số thực khả vi cấp 2 sao cho $\sup_{x \in \mathbb{R}} |f(x)| \le 1 \;\;, \sup_{x \in \mathbb{R}} |f''(x)| \le 1 $

Chứng minh : $$ \sup_{x \in \mathbb{R}} |f'(x)| \le \sqrt{2} $$

Mạnh hơn nữa, thay giả thiết $\sup_{x \in \mathbb{R}} |f''(x)| \le 1$ bởi $\dfrac{|f'(x)-f'(y)|}{x-y} \le 1 \;, \forall x,y \in \mathbb{R} ,\; x>y$



#384433 Ôn thi Olympic Toán học sinh viên 2015 [Giải tích]

Posted by phudinhgioihan on 07-01-2013 - 17:24 in Thảo luận về các kì thi, các kì kiểm tra Toán sinh viên

Tiếp tục chủ đề :ukliam2:

Bài 6: Cho $f:\left[0;1\right]\to\mathbb{R} $ là hàm có đạo hàm liên tục thỏa mãn
$\displaystyle\int_0^1f\left(x\right)dx= \displaystyle\int_0^1xf\left(x\right)dx$ .Chứng minh rằng tồn tại $c\in\left(0,1\right)$ sao cho $$f\left(c\right)=\displaystyle f'\left(c\right)\int_0^cf\left(x\right)dx$$



#390885 Ôn thi Olympic Toán học sinh viên 2015 [Giải tích]

Posted by phudinhgioihan on 27-01-2013 - 20:06 in Thảo luận về các kì thi, các kì kiểm tra Toán sinh viên

Chợt nhớ một bài sáng tác cách đây hai năm :D

Bài 24: Gọi $D$ là tập các hàm số liên tục từ $[0;1]$ vào $[0;1]$ .

Tìm $n \in \mathbb{N}^*$ sao cho $\forall f \in D , \exists x_0 \in [0;1]$ thỏa $nf(x_0)=(1-x_0) \left[ f(x_0)+f(1-x_0) \right] $



#391759 Ôn thi Olympic Toán học sinh viên 2015 [Giải tích]

Posted by phudinhgioihan on 30-01-2013 - 17:39 in Thảo luận về các kì thi, các kì kiểm tra Toán sinh viên

Bài 9:

Ta có:

\[I_n = \int\limits_0^1 {\frac{{nf\left( x \right)dx}}{{1 + {n^2}{x^2}}}} = \int\limits_0^1 {f\left( x \right)d\left( {\arctan nx} \right)} = \left. {f\left( x \right).\arctan nx} \right|_0^1 - \int\limits_0^1 {\arctan nx.f'(x)dx} \]

-------------------------------------------------------------------------------------

Anh Đạt chắc đánh nhầm thôi. :D.


Gõ nhầm thiệt, đúng là $n \to +\infty$

@Việt: Bài giải tưởng chừng đúng nhưng lại sai từ dòng đầu tiên!

Lý do: Chỉ có giả thiết $f$ liên tục trên $[0;1]$ nên chưa chắc nó khả vi, do đó không thể đi bằng con đường qua $f'$ được. Hơn nữa, em xem lại định lý trung bình tích phân loại 2, điều kiện ở đây không thể sử dụng được, kể cả có tồn tại $f'$ hay không.

@ablrise: Ở quê tối muỗi quá nên chắc không post lời giải được, hẹn ngày mai vậy.

Tặng mọi người bài mới sáng tác lúc sáng:

Bài 27:

Cho $f: [0;1] \to \mathbb{R}$ khả vi liên tục và giảm trên $[0;1]$. Chứng minh

$$f(x) \le \int_0^1 f(t)dt+\dfrac{1}{2} \int_0^1 f'^2(t)dt+\dfrac{(1-x)^3}{6}$$

Chặn trên bởi một đa thức ^^. Khó hơn tí, tìm trường hợp đẳng thức :D



#393472 Ôn thi Olympic Toán học sinh viên 2015 [Giải tích]

Posted by phudinhgioihan on 05-02-2013 - 19:59 in Thảo luận về các kì thi, các kì kiểm tra Toán sinh viên

Bài 31: (mới chế :D)

Cho $[a;b] \subset \mathbb{R}$, $f:[a;b] \to \mathbb{R}$ có đạo hàm cấp 2 liên tục sao cho $f(a)=f(b)=f'(a)=f'(b)=0\;, f''(x) \ge 0 \;\forall x \in [a;b]$

Chứng minh tồn tại $c \in [a;b] $ sao cho $\dfrac{(b-a)^3}{6} f''( c ) \ge \int_a^b f(x)dx \ge \dfrac{(b-a)^3}{24}f''( c )$



#393304 Ôn thi Olympic Toán học sinh viên 2015 [Giải tích]

Posted by phudinhgioihan on 05-02-2013 - 07:17 in Thảo luận về các kì thi, các kì kiểm tra Toán sinh viên

Quen nhề :-"

Câu dưới tương tự :P


Hai bdt này đã có từ tận 1930!
Mục đích đưa ra là để cải tiến chứng minh đơn giản tí, lời giải cũ khủng quá @@
$$\int_0^a |f(x)f'(x)|dx \le \int_0^a |f'(x)| \int_0^x |f'(t)|dt dx $$
$$ \le \int_0^a \int_0^x |f'(t)|dt d\left(\int_0^x |f'(t)|dt \right)$$
$$\le \frac{1}{2} \left(\int_0^a |f'(x)|dx \right)^2 \le \frac{1}{2} \int_0^a dx \int_0^a f'^2(x)dx$$
$$\le \frac{1}{2} \int_0^a f'^2(x)dx $$
Bất đẳng thức thứ hai:
Ta có: $$\int_0^a |f(x)f'(x)|dx=\int_0^{\frac{a}{2}} |f(x)f'(x)|dx+\int_{\frac{a}{2}}^a |f(x)f'(x)|dx$$
** $\int_0^{\frac{a}{2}} |f(x)f'(x)|dx \le \frac{a}{4} \int_0^{\frac{a}{2}}f'^2(x)dx $
** Đặt $g(a-x)=f(x)$ , dễ thấy $g$ có các tính chất của $f$, do đó
$$\int_0^{\frac{a}{2}} |g(x)g'(x)|dx \le\frac{a}{4} \int_0^{\frac{a}{2}} g'^2(x)dx $$
$$\Leftrightarrow \int_0^{\frac{a}{2}} |f(a-x)f'(a-x)|dx \le \frac{a}{4} \int_0^{\frac{a}{2}} f'^2(a-x)dx $$
$$\Leftrightarrow \int_{\frac{a}{2}}^a |f(x)f'(x)|dx \le \frac{a}{4} \int_{\frac{a}{2}}^a f'^2(x)dx $$
Cộng hai bdt lại ta có đpcm.



#392487 Ôn thi Olympic Toán học sinh viên 2015 [Giải tích]

Posted by phudinhgioihan on 02-02-2013 - 15:07 in Thảo luận về các kì thi, các kì kiểm tra Toán sinh viên

Bài 29:

Cho $f:[a;b] \to \mathbb{R}$ (với $[a;b] \subset \mathbb{R}$) không là hàm tuyến tính và có đạo hàm cấp hai trên $(a;b)$. Chứng minh $f(x)=0$ vô nghiệm trên $(a;b)$ nếu tồn tại $c \in (a;b)$ sao cho

$$ f'^2( c ) -2f ( c)f''(x) <0 \;\;, \forall x \in (a;b)$$




#392477 Ôn thi Olympic Toán học sinh viên 2015 [Giải tích]

Posted by phudinhgioihan on 02-02-2013 - 13:16 in Thảo luận về các kì thi, các kì kiểm tra Toán sinh viên

Bài 28:Cho $f$ là hàm liên tục trên $\mathbb{R}$. Đặt $$f_1(x)=\int_0^xf(t)dt,...,f_n(x)=\int_0^xf_{n-1}(t)dt.$$ Chứng minh rằng $$f_{n+1}(x)=\frac{1}{n!}\int_0^x(x-t)^nf(t)dt,n\ge 1$$


Với chú ý:

$$ f_{n+1}(x)=\int_0^x \int_0^t f_n(u)du dt =\int_0^x \int_0^t f_n(u)du d(t-x)$$

$$=(t-x) \int_0^t f_n(u)du |_0^x -\int_0^x (t-x) f_n(t)dt =\int_0^x (x-t)f_n(t)dt $$

$$=\int_0^x (x-t) \int_0^t f_{n-1}(u)du dt =-\dfrac{1}{2} \int_0^x \int_0^t f_{n-1}(u)du d((x-t)^2)$$

$$=-\dfrac{1}{2} (x-t)^2 \int_0^t f_{n-1}(u)du |_0^x +\dfrac{1}{2} \int_0^x (x-t)^2f_{n-1}(t)dt$$

$$=\dfrac{1}{2} \int_0^x (x-t)^2f_{n-1}(t)dt $$

Tương tự, sử dụng liên tiếp tích phân từng phần ta được

$$f_{n+1}(x)=\dfrac{1}{n!} \int_0^x (x-t)^n f(t)dt$$



#393478 Ôn thi Olympic Toán học sinh viên 2015 [Giải tích]

Posted by phudinhgioihan on 05-02-2013 - 20:05 in Thảo luận về các kì thi, các kì kiểm tra Toán sinh viên

Bài 32: (mới chế :D)

Cho $f:[0;2] \to [0;\dfrac{1}{2} ]$ đơn điệu tăng và có đạo hàm cấp 2 liên tục trên $[0;2]$ đồng thời $f'(0).f'(1)=1$. Chứng minh tồn tại $c \in [0;2]$ sao cho

$$f''( c) \ge 3 \int_0^2 f(x)dx $$